Calculating Lie Derivative for Metric Tensor with Given Coordinates and Vector

Click For Summary
The discussion focuses on calculating the Lie derivative of a given metric tensor using specific coordinates and a vector field. The metric tensor is defined as g_{ab}=diag(-(1-\frac{2M}{r}),1-\frac{2M}{r},r^2,R^2sin^2\theta), and the vector field is represented by E^i=\delta^t_0, indicating a constant value in the time component. The participants clarify that the derivatives of the last two terms in the Lie derivative formula vanish due to the constancy of the vector field. The main confusion arises around the interpretation of \delta^t_0, which is confirmed to represent the time component being 1 at t=0 and zero otherwise. The conclusion is that the calculation of the Lie derivative simplifies significantly under these conditions.
trv
Messages
74
Reaction score
9

Homework Statement



Calculate the lie derivative of the metric tensor, given the metric,

<br /> g_{ab}=diag(-(1-\frac{2M}{r}),1-\frac{2M}{r},r^2,R^2sin^2\theta)<br />

and coordinates (t,r,theta,phi)

given the vector

<br /> E^i=\delta^t_0<br />





Homework Equations



<br /> (L_Eg)ab=E^cd_cg_{ab}+g_{cb}d_aE^c+g_{ac}d_bE^c<br />


The Attempt at a Solution



<br /> (L_Eg)ab=E^cd_cg_{ab}+g_{cb}d_aE^c+g_{ac}d_bE^c<br />

all derivatives above being partial

Now the Last two terms go to zero, since E^i=Kronecker delta=constant and so its derivative is zero.

So,
<br /> (L_Eg)ab=E^cd_cg_{ab}<br />

<br /> (L_Eg)ab=\delta^t_0 d_cg_{ab}<br />

I'm unsure how to take it from here.

Firstly, I'm unsure what
<br /> \delta^t_0<br />

means. Does it means we get the result 1 at t=0 and zero for all other times?

How does it then affect the equation below.

<br /> (L_Eg)ab=\delta^t_0 d_cg_{ab}<br />

Please help.
 
Last edited:
Physics news on Phys.org
anyone?
 
I think what they meant to write is E^i=\delta^i_0 i.e. the vector field with a constant value 1 in the time component. So the Lie derivative is pretty trivial.
 
Oh ok, so the 0 stands for the time component. That makes sense.
 
Question: A clock's minute hand has length 4 and its hour hand has length 3. What is the distance between the tips at the moment when it is increasing most rapidly?(Putnam Exam Question) Answer: Making assumption that both the hands moves at constant angular velocities, the answer is ## \sqrt{7} .## But don't you think this assumption is somewhat doubtful and wrong?

Similar threads

  • · Replies 1 ·
Replies
1
Views
2K
  • · Replies 3 ·
Replies
3
Views
2K
  • · Replies 71 ·
3
Replies
71
Views
2K
  • · Replies 4 ·
Replies
4
Views
1K
  • · Replies 1 ·
Replies
1
Views
978
  • · Replies 11 ·
Replies
11
Views
2K
Replies
14
Views
4K
Replies
2
Views
1K
  • · Replies 8 ·
Replies
8
Views
2K
  • · Replies 7 ·
Replies
7
Views
3K